Feb 17

设$a,b,c\geq 0$,$a+b+c=3$,证明
\[ \frac{a}{\sqrt{b^2+b+1}}+\frac{b}{\sqrt{c^2+c+1}}+\frac{c}{\sqrt{a^2+a+1}}\geq \sqrt{3} \]
证明:
齐次化后就是
\[ \sum\frac{a}{\sqrt{a^2+13b^2+c^2+5ab+5bc+2ac}}\geq \frac{\sqrt{3}}{3} \]
由Holder
\begin{align*}
&\left(\sum\frac{a}{\sqrt{a^2+13b^2+c^2+5ab+5bc+2ac}}\right)^{2}\left(\sum a(a+c)^3(a^2+13b^2+c^2+5ab+5bc+2ac) \right)\\
&\geq \left(\sum{a^2}+\sum{ab}\right)^3
\end{align*}
所以只要证明
\[ 3\left(\sum{a^2}+\sum{ab}\right)^3\geq \left(\sum a(a+c)^3(a^2+13b^2+c^2+5ab+5bc+2ac) \right) \]
就是
\[ 2\sum{a^6}+3\sum{a^5b}+4\sum{ab^5}+2\sum{a^2b^3c}+2\sum{a^4bc}-14\sum{ab^3c^2}+11\sum{a^3b^3}+8\sum{a^2b^4}-54a^2b^2c^2\geq 0\]
这个成立,由于
\[ 2\sum{a^2b^3c}+2\sum{a^4bc}\geq 4\sum{a^3b^2c} \]
由AM-GM
\[ \frac{72}{126}a^5b+\frac{36}{126}b^5c+\frac{18}{126}c^5a\geq a^3b^2c\]
得到
\[ \sum{a^5}b\geq \sum{a^3b^2c}\]
同时
\[ \frac{4}{6}a^4b^2+\frac{1}{6}b^4c^2+\frac{1}{6}c^4a^2\geq a^3b^2c \]
得到
\[ \sum{a^4b^2}\geq \sum{a^3b^2c}\]
显然又有
\[ \sum{a^3b^3}\geq \sum{a^3b^2c} \]
这样
\[2\sum{a^6}+3\sum{a^5b}+4\sum{ab^5}+2\sum{a^2b^3c}+2\sum{a^4bc}+11\sum{a^3b^3}+8\sum{a^2b^4}\geq 32\sum{a^3b^2c}\]
Done!

________________________________________________________________________________

西神后来指出:注意到局部(切线法)

\[ \frac{1}{\sqrt{x^2+x+1}}\geq \frac{\sqrt{3}}{2}-\frac{x}{2\sqrt{3}}\]

就有

\[ LHS\geq \frac{\sqrt{3}}{2}(a+b+c)-\frac{ab+bc+ca}{2\sqrt{3}} \]

 

Feb 17

Let $a,b,c>0$,with $a+b+c=3$,Prove that
\[ \frac{1}{(a+b)(4-ab)}+\frac{1}{(b+c)(4-bc)}+\frac{1}{(c+a)(4-ca)}\geq \frac{1}{2} \]
证明:由Cauchy-Schwarz
$$ \sum_{cyc}\frac{1}{(a+b)(4-ab)}=\sum_{cyc}\frac{(3c+2a+2b)^2}{(3c+2a+2b)^2(a+b)(4-ab)}\geq\frac{49(a+b+c)^2}{\sum\limits_{cyc}(3c+2a+2b)^2(a+b)(4-ab)} $$
所以,只要证明
$$ 98(a+b+c)^5\geq3\sum\limits_{cyc}(3c+2a+2b)^2(a+b)(4(a+b+c)^2-9ab) $$
就是
\[ 2\sum{a^5}+10\sum{a^4(b+c)}+20\sum{a^3(b^2+c^2)}\geq 30abc\sum{ab}+32abc\sum{a^2}\]
由Murihead 显然。

Dec 7
设$n$阶矩阵$A,B$可交换,证明:$$r(A+B)\leq r(A)+r(B)-r(AB)$$
证明:先证一个引理:
设$A,B$是数域$K$上两个$n\times n$矩阵且$AB=BA$.又设

\[ C=\left[
\begin{array}{cc}
A \\
B \\
\end{array}\right]\]

则有

\[ r(A)+r(B)\geq r(C)+r(AB) \]
证明:考察奇次线性方程组$AX=0$和$BX=0$,它们的全部公共解向量恰好为$CX=0$的解向量。故设$CX=0$的一个基础解系是
\[ \varepsilon_{1},\varepsilon_{2},\cdots\varepsilon_{r}\]
这里$r=n-r(C)$,我们知道,它可以分别扩充到$AX=0$的一个基础解系
\[ \varepsilon_{1},\varepsilon_{2},\cdots\varepsilon_{r},\eta_{1},\eta_{2}\cdots,\eta_{s}\]
和$BX=0$的一个基础解系
\[ \varepsilon_{1},\varepsilon_{2},\cdots\varepsilon_{r},\omega_{1},\omega_{2}\cdots,\omega_{t}\]
这里$r+s=n-r(A),r+t=n-r(B)$.而线性组合
\[ a_{1}\eta_{1}+\cdots+a_{s}\eta_{s}\]
是$AX=0$的解向量,但当它非零时它不是$BX=0$的解向量,否则它就是两者的公共解向量,即有$CX=0$,从而可以被$CX=0$的解系线性表示
\[ a_{1}\eta_{1}+\cdots+a_{s}\eta_{s}=k_{1}\varepsilon_{1}+\cdots k_{r}\varepsilon_{r}\]
此时由于$\varepsilon_{1},\varepsilon_{2},\cdots\varepsilon_{r},\eta_{1},\eta_{2}\cdots,\eta_{s}$线性无关,所以得到$a_{1}=a_{2}=\cdots a_{s}=k_{1}=\cdots k_{r}=0$,这与$a_{1}\eta_{1}+\cdots+a_{s}\eta_{s}$非零矛盾,由此可见
\[ \varepsilon_{1},\varepsilon_{2},\cdots\varepsilon_{r},\eta_{1},\eta_{2}\cdots,\eta_{s},\omega_{1},\omega_{2}\cdots,\omega_{t}\]
线性无关。
现在,上面都是$ABX=BAX=0$的解向量,它们又线性无关,故$r+s+t\leq n-r(AB)$即
\[ (r+s)+(r+t)-r=n-r(A)+n-r(B)-(n-r(C))\leq n-r(AB)\]
\[ \Leftrightarrow r(C)+r(AB)\leq r(A)+r(B) \]
引理证明完毕!
 
最后回到原题,只要注意到

\[A + B = \left[{{E_n},{E_n}}\right]\left[ {\begin{array}{*{20}{c}}
A\\
B
\end{array}} \right]\]

\[ r(A+B)\leq r(C)\]
故马上有
\[ r(A+B)\leq r(A)+r(B)-r(AB) \]
Done!
Dec 1
(1)设$\displaystyle \sum_{n=1}^{\infty}a_n$是正项收敛级数,试证明
\[ I=\sum_{n=1}^{\infty}a_{n}^{1-\frac{1}{n}}\]
收敛。
(2)设$\displaystyle\sum_{n=1}^{\infty}a_n$是正项收敛级数,试证明
\[ I=\sum_{n=1}^{\infty}a_{n}^{\frac{bn}{1+bn}}\qquad (b>0) \]
(3)设正项级数$\displaystyle\sum_{n=1}^{\infty}a_n$收敛。若正数列$\{b_n\}$满足$\displaystyle b_{n}=o\left(\frac{1}{\ln n}\right)(n\to\infty)$,则
\[ I=\sum_{n=2}^{\infty}a_{n}^{1-b_n}\]
收敛。
(4)设$\displaystyle\sum_{n=1}^{\infty}a_n$是收敛的正项级数,则
\[ \sum_{n=1}^{\infty}a_n\cdot\frac{\ln\left(\frac{1}{a_n}\right)}{\ln(1+a_n)}\]
收敛。
(5)设$0<a_n<1$,$n\in \mathbf{N}$,若$\displaystyle\sum_{n=1}^{\infty}\frac{a_n}{\ln a_n}$收敛,则
\[ \sum_{n=1}^{\infty}\frac{a_n}{\ln(n+1)}\]
收敛。
(6)设$\{a_n\}$是递增的正数列。若$\displaystyle\sum_{n=1}^{\infty}\frac{1}{a_n}$收敛,则对任意的自然数$k$,级数$\displaystyle\sum_{n=1}^{\infty}\frac{(\ln a_n)^k}{a_n}$收敛的充分必要条件是级数$\displaystyle\sum_{n=1}^{\infty}\frac{(\ln n)^k}{a_n}\displaystyle$收敛.
______________________________________________________________________________________________
 
 
Nov 19

设$x,y,z>0$,且满足$\displaystyle \sum_{cyc}\frac{1}{1+x}=1$.我们有
\[ \sum_{cyc}\frac{x}{\sqrt{yz}(x+1)}\geq 1 \]
证明:设$$a=\frac{1}{1+x},b=\frac{1}{1+y},c=\frac{1}{1+z}$$
\[ \Rightarrow x=\frac{b+c}{a},y=\frac{c+a}{b},z=\frac{a+b}{c} \]
齐次化后
\[ \Leftrightarrow \sum(b+c)\sqrt{bc(b+c)}\geq (a+b+c)\sqrt{(a+b)(b+c)(c+a)}\]
两边平方,化简成
\[ \sum{bc(b+c)^3}+2\sum c(c+b)(c+a)\sqrt{ab(a+c)(b+c)}\geq (a+b+c)^2(a+b)(b+c)(c+a) \]
Or
\[ \sum c(c+b)(c+a)\sqrt{ab(a+c)(b+c)}\geq abc\left(3\sum{a^2}+5\sum{ab}\right)\]
Or
\[ \sum (c+b)(c+a)\sqrt{\frac{(a+c)(b+c)}{ab}} \geq \left(3\sum{a^2}+5\sum{ab}\right)\]
Cauchy-Schwarz
\[ \sqrt{(a+c)(b+c)}\geq c+\sqrt{ab}\Rightarrow \sqrt{\frac{(a+c)(b+c)}{ab}}\geq 1+\frac{c}{\sqrt{ab}}\]
所以只要证明
\[ \sum (c+b)(c+a)\left(1+\frac{c}{\sqrt{ab}}\right)\geq \left(3\sum{a^2}+5\sum{ab}\right)\]
\[ \Leftrightarrow \sum (c+b)(c+a)\cdot \frac{c}{\sqrt{ab}}\geq 2\sum{a^2}+2\sum{ab}\]
用$a^2,b^2,c^2$替换$a,b,c$得到
\[ \sum{a^7}+\sum{a^5(b^2+c^2)}+\sum{a^3b^2c^2}\geq 2\sum{a^5bc}+2abc\cdot\sum{a^2b^2}\]
注意到
\[ \sum{a^5(b^2+c^2)}-2\sum{a^5bc}=\sum{a^5(b-c)^2}\geq 0 \]
\[\sum{a^7}+\sum{a^3b^2c^2}\geq 2\sum{a^5bc}\geq 2abc\sum{a^2b^2}\]
故不等式得证。
\[ \sqrt{2}\geq P=\sum_{cyc}\frac{(b+c)\sqrt{bc(b+c)}}{\sqrt{(a+b)(b+c)(c+a)}(a+b+c)}\geq 1  \]

另外我们有
\[ \sum_{cyc}\frac{x}{\sqrt{yz}(x+1)}\leq \sqrt{2} \]
作替换
\[ x=\frac{ab+ac}{bc},y=\frac{bc+ba}{ac},z=\frac{ca+cb}{ab}\]
于是,不等式变成
\[ \frac{a^2(b+c)}{\sqrt{(a+b)(a+c)}}+\frac{b^2(c+a)}{\sqrt{(b+c)(b+a)}}+\frac{c^2(a+b)}{(c+a)(c+b)}\leq \sqrt{2}(ab+bc+ca) \]
Or
\[ \sum_{cyc} a^2(b+c)\sqrt{b+c}\leq \sqrt{2(a+b)(b+c)(c+a)}\cdot(ab+bc+ca)\]
注意到Cauchy-Schwarz
\begin{align*}
\sum_{cyc} a^2(b+c)\sqrt{b+c}=\sum_{cyc}\sqrt{a^4(b+c)^3}&\leq \sqrt{[a(b+c)+b(c+a)+c(a+b)][a^3(b+c)^2+b^3(c+a)^2+c^3(a+b)^2]}\\
&=\sqrt{2(ab+bc+ca)[a^3(b+c)^2+b^3(c+a)^2+c^3(a+b)^2]}
\end{align*}
故只要证明
\[ (a+b)(b+c)(c+a)(ab+bc+ca)\geq a^3(b+c)^2+b^3(c+a)^2+c^3(a+b)^2\]
展开显然。